Given that f(x)=2x+1 and g(x)=-5x+2 slove for f(g(x)) when x=3

Answers

Answer 1
f(g(x))=2(-5x+2)+1= -10x+5
f(g(3))= -10(3)+5= -25

So f(g(x)) when x=3 is -25
Answer 2

Answer:

The answer is - 91

Step-by-step explanation:

f(x)=2x+1

g(x)=-5x+2

To find f(g(3)) first find f(g(x))

To find f(g(x)) multiply f(x) by g(x)

that's

f(g(x)) = (2x + 1)( - 5x + 2)

f(g(x)) = - 10x² + 4x - 5x + 2

f(g(x)) = - 10x² - x + 2

To find f(g(x)) when x = 3 substitute 3 into

f(g(x))

That's

f(g(3)) = -10(3)² - 3 + 2

f(g(3)) = -90 - 3 + 2

We have the final answer as

f(g(3)) = - 91

Hope this helps you


Related Questions

PLEASE HELP, WILL GIVE BRAINLIEST IF CORRECT!!!! (08.06 MC) Mike and his friends bought cheese wafers for $2 per packet and chocolate wafers for $1 per packet at a carnival. They spent a total of $25 to buy a total of 20 packets of wafers of the two varieties. Part A: Write a system of equations that can be solved to find the number of packets of cheese wafers and the number of packets of chocolate wafers that Mike and his friends bought at the carnival. Define the variables used in the equations. (5 points) Part B: How many packets of chocolate wafers and cheese wafers did they buy? Explain how you got the answer and why you selected a particular method to get the answer. (5 points)

Answers

Answer:

x = 5 , y = 15

Step-by-step explanation:

You can solve this using substitution.

Let the quantity of cheese wafers be denoted by x and the quantity of chocolate wafers denoted by y

2x + 1y = 25

x + y = 20

These two equations are the answer to part A, (remember to include the above prompt which says what x and y denote).

For part B I used substitution because it was more applicable to the question then addition or elimination.

ACTUAL WORK

Set 2x + 1y = 25 equal to x

x = 25 - y / 2

Replace x with y in the second equation

(25 - y / 2) + y = 20

And solve for y

y = 15

Since we know what y is we can replace y in the second equation and find what x is

x + 15 = 20

Solve for x

x = 5

Answer:

5 Cheese Wafers and 15 Chocolate Wafers

Step-by-step explanation:

The graph of f(x) = StartRoot x EndRoot is reflected over the y-axis. Use the graphing calculator to graph this reflection. Which list contains three points that lie on the graph of the reflection? (–81, 9), (–36, 6), (–1, 1) (1, –1), (16, –4), (36, –6) (–49, 7), (–18, 9), (–1, 1) (1, –1), (4, –16), (5, –25)

Answers

Answer:

(–81, 9), (–36, 6), (–1, 1) are the correct three points.

Step-by-step explanation:

Given the function:

[tex]f(x) =\sqrt x[/tex]

Please refer to the attached image.

The green line shows the graph of actual function.

It is reflected over y axis.

The reflected graph is shown in black color in attached image.

When reflected over y axis, the sign of variable [tex]x[/tex] changes from Positive to Negative.

So, the resultant function becomes:

[tex]f(x)=\sqrt{-x}[/tex]

i.e. we will have to give the values of x as negative now.

so, the options in which value of x is negative are the possible answers only.

The possible answers are:

(–81, 9), (–36, 6), (–1, 1) and

(–49, 7), (–18, 9), (–1, 1)

Now, we will check the square root function condition.

In the 2nd option, (–18, 9) does not satisfy the condition.

So, the correct answer is:

(–81, 9), (–36, 6), (–1, 1)

Answer:

A on E2020

Step-by-step explanation:

:)

need help will give 5 stars.

Answers

Answer:

t=0.64

Step-by-step explanation:

h = -16t^2 +4t +4

We want h =0 since it is hitting the ground

0 = -16t^2 +4t +4

Using the quadratic formula

a = -16  b = 4  c=4

-b ± sqrt( b^2 -4ac)

----------------------------

         2a

-4 ± sqrt( 4^2 -4(-16)4)

----------------------------

         2(-16)

-4 ± sqrt( 16+ 256)

----------------------------

         -32

-4 ± sqrt( 272)

----------------------------

         -32

-4 ± sqrt( 16*17)

----------------------------

         -32

-4 ± sqrt( 16) sqrt(17)

----------------------------

         -32

-4 ± 4 sqrt(17)

----------------------------

         -32

Divide by -4

1 ±  sqrt(17)

----------------------------

         8

To the nearest hundredth

t=-0.39

t=0.64

Since time cannot be negative

t=0.64

Answer:

0.64  

Step-by-step explanation:

0 = -16t^2 + 4t + 4

-4(4t^2 - t -1) = 0

t = [-(-1) +/- sqrt (1 - 4*4*-1)] / 8)

t = 0.64, -0.39

answer is 0.64

Need help please asap this is not asap but please still give an answer im stuck

Answers

Answer:

135 cubes

Step-by-step explanation:

First, find the volume of the box with the equation V = Bh, where B is the area of the base and h is the height.

V = (2.25)(0.75)(1.25)

V = 2.109375

Next, find the volume of one cube with the side length 1/4 with V = Bh:

V = (0.25)(0.25)(0.25)

V = 0.015625

Then, divide the volume of the box by the volume of one cube:

2.109375 / 0.015625

= 135

Jenny had a wardrobe full of 35 different shirts. In order to make more space in her closet, she got rid of 9 of them. What is a reasonable
estimate for the percentage of shirts Jenny got rid of?

Answers

There is no one set answer because there are many ways to estimate here.

35 rounds to 40

9 rounds to 10

She got rid of 10 shirts out of 40, so 10/40 = 1/4 = 0.25 = 25% is the estimated percentage of shirts she got rid of. This is one possible estimate.

Using a calculator, the actual percentage is 9/35 = 0.2571 = 25.71% approximately. So our estimate isn't too bad. Our estimate is an underestimate.

A bag contains twelve marbles, which includes seven red marbles and five blue marbles. Roja reaches into the bag and pulls out four marbles. a) How many different sets of four marbles can be pulled from this bag? b) How many of these sets contain two red marbles and two blue marbles? c) How many of these sets contain all red marbles? d) How many of these sets contain all red marbles or all blue marbles?

Answers

Answer:

a) 495

b) 210

c) 35

d) 40

Step-by-step explanation:

Given a total of 12 marbles.

n = 12

Number of red marbles = 7

Number of blue marbles = 5

a) Number of different sets of 4 marbles that can be made from this bag ?

This is a simple combination problem.

where n = 12 and r = 4.

So, answer will be:

[tex]_{12}C_4[/tex]

Formula:

[tex]_{n}C_r = \dfrac{n!}{(n-r)!r!}[/tex]

[tex]_{12}C_4 = \dfrac{12!}{(8)!4!} = \dfrac{12\times 11\times 10\times 9}{4 \times 3\times 2} =\bold{495}[/tex]

b) Two red and two blue marbles:

The answer will be:

[tex]_{7}C_2 \times _{5}C_2 = \dfrac{7\times 6}{2} \times \dfrac{5\times 4}{2} =\bold{210}[/tex]

c) all red marbles.(4 chosen out of 7 red and 0 chosen out of 5 blue marbles)

[tex]_{7}C_4 \times _{5}C_0 = \dfrac{7\times 6\times 5\times 4}{4\times 3\times 2} =\bold{35}[/tex]

d) all red or all blue.(all red marbles plus all blue marbles)

All red marbles:

[tex]_{7}C_4 \times _{5}C_0 = \dfrac{7\times 6\times 5\times 4}{4\times 3\times 2} \times 1=\bold{35}[/tex]

All blue marbles:

[tex]_{7}C_0 \times _{5}C_4 = 1 \times \dfrac{5\times 4\times 3\times 2}{4\times 3\times 2} =\bold{5}[/tex]

So, answer is 40.

A rectangular prism has a volume of 864 cubic units. How many cubic unit will fill the volume of the solid if they were packed without any gaps or overlaps

Answers

Answer: 864.

Step-by-step explanation:

The volume of a rectangular prism has a volume equal to:

V = W*L*H

W = width

L = length

H = height

We know that the volume is equal to 864 cubic units.

This means that if we want to fill the prism such that there is no gap or overlap, we should use exactly 864 unit cubes.

The height of a building model is 2% of its actual height. If the building
model is 3 feet tall, how tall is the actual building?

Answers

Answer:

x = 150 feets

Step-by-step explanation:

Given that,

The height of a building model is 2% of its actual height.

The building model is 3 feet tall, h = 3 feet

We need to find the height of the actual building. Let it is x.

According to question,

h = 2% of x

We have, h = 3 feet

So,

[tex]x=\dfrac{h}{2\%}\\\\x=\dfrac{3}{2/100}\\\\x=150\ \text{feet}[/tex]

So, the actual height of the building is 150 feets.

If sin Θ = 5 over 6, what are the values of cos Θ and tan Θ?

Answers

Answer:

Check explanation

Step-by-step explanation:

Sin∅=5/6

Opp=5. Hyp=6

Adj= (√6²+5²)

= √11

Cos∅=(√11)/6

Tan∅=5/(√11)

Answer: cos(theta) = sqrt(11)/6 ; tan(theta)=5/sqrt(11).

Explanation:
sin inverse is restricted to the first and fourth quadrant, so the other side of the triangle is positive sqrt(11) by the Pythagorean theorem.

Solve. 4x−y−2z=−8 −2x+4z=−4 x+2y=6 Enter your answer, in the form (x,y,z), in the boxes in simplest terms. x= y= z=

Answers

Answer:

(-2, 4, 2)

Where x = -2, y = 4, and z = 2.

Step-by-step explanation:

We are given the system of three equations:

[tex]\displaystyle \left\{ \begin{array}{l} 4x -y -2z = -8 \\ -2x + 4z = -4 \\ x + 2y = 6 \end{array}[/tex]

And we want to find the value of each variable.

Note that both the second and third equations have an x.

Therefore, we can isolate the variables for the second and third equation and then substitute them into the first equation to make the first equation all one variable.

Solve the second equation for z:

[tex]\displaystyle \begin{aligned} -2x+4z&=-4 \\ x - 2 &= 2z \\ z&= \frac{x-2}{2}\end{aligned}[/tex]

Likewise, solve the third equation for y:

[tex]\displaystyle \begin{aligned} x+2y &= 6\\ 2y &= 6-x \\ y &= \frac{6-x}{2} \end{aligned}[/tex]

Substitute the above equations into the first:

[tex]\displaystyle 4x - \left(\frac{6-x}{2}\right) - 2\left(\frac{x-2}{2}\right)=-8[/tex]

And solve for x:

[tex]\displaystyle \begin{aligned} 4x+\left(\frac{x-6}{2}\right)+(2-x) &= -8 \\ \\ 8x +(x-6) +(4-2x) &= -16 \\ \\ 7x-2 &= -16 \\ \\ 7x &= -14 \\ \\ x &= -2\end{aligned}[/tex]

Hence, x = -2.

Find z and y using their respective equations:

Second equation:

[tex]\displaystyle \begin{aligned} z&=\frac{x-2}{2} \\ &= \frac{(-2)-2}{2} \\ &= \frac{-4}{2} \\ &= -2\end{aligned}[/tex]

Third equation:

[tex]\displaystyle \begin{aligned} y &= \frac{6-x}{2}\\ &= \frac{6-(-2)}{2}\\ &= \frac{8}{2}\\ &=4\end{aligned}[/tex]

In conclusion, the solution is (-2, 4, -2)

Answer:

x = -2

y =4

z=-2

Step-by-step explanation:

4x−y−2z=−8

−2x+4z=−4

x+2y=6

Solve the second equation for x

x = 6 -2y

Substitute into the first two equations

4x−y−2z=−8

4(6-2y) -y -2 = 8  

24 -8y-y -2z = 8

-9y -2z = -32

−2(6-2y)+4z=−4

-12 +4y +4z = -4

4y+4z = 8

Divide by 4

y+z = 2

z =2-y

Substitute this into -9y -2z = -32

-9y -2(2-y) = -32

-9y -4 +2y = -32

-7y -4 = -32

-7y =-28

y =4

Now find z

z = 2-y

z = 2-4

z = -2

Now find x

x = 6 -2y

x = 6 -2(4)

x =6-8

x = -2

Tim has an after-school delivery service that he

provides for several small retailers in town. He

uses his bicycle and charges $1.25 for a delivery

made within 1 mi, $1.70 for a delivery of at

least 1 mi but less than 1 mi, $2.15 for a

delivery of at least 1. mi but less than 2 miles,

and so on. If Tim raised his rates by 10%, what

would he be paid to deliver a package 35

miles?

Answers

Answer:

Step-by-step explanation:

tim has an after school delivery service that he provides for several small retailers in town. he uses his bicycle and charges $1.25 for a delivery made within 1 1/2 miles, $1.70 for a delivery of at least 1 1/2 miles but less than 1 3/4 miles. $2.15 for a delivery of at least 1 3/4 miles but less than 2 miles, and so on. if tim raised his rates by 10%, what would he be paid to deliver a package 3 1/8 miles.

Answer:

From the question asked the cost of additional 1/4 mile (1 3/4 - 1 1/2) is $0.45 ($1.7 - $1.25). If the rate is increased by 10% (0.1), the new price for an additional 1/4 mile would be 1.1 (1 + 0.1) × 0.45 = $0.495.

Tim new charge rate are as follows:

$1.25 for a delivery made within 1 1/2 miles

$1.745 for a delivery of at least 1 1/2 miles but less than 1 3/4

$2.24 or a delivery of at least 1 3/4 miles but less than 2

$2.735 or a delivery of at least 2 miles but less than 2 1/4

$3.23 or a delivery of at least 2 1/4 miles but less than 2 1/2

$3.725 or a delivery of at least 2 1/2 miles but less than 2 3/4

$4.22 or a delivery of at least 2 3/4 miles but less than 3

$4.715 or a delivery of at least 3 miles but less than 3 1/4

Since 3 1/8 is within 3 miles and 3 1/4 miles, Tim would charge $4.715 to deliver a package 3 1/8 miles.

 

A combination lock uses three numbers between 1 and 46 with​ repetition, and they must be selected in the correct sequence. Is the name of​ "combination lock"​ appropriate? Why or why​ not? Choose the correct answer below. A. ​No, because the multiplication counting rule would be used to determine the total number of combinations. B. ​Yes, because the combinations rule would be used to determine the total number of combinations. C. ​No, because factorials would be used to determine the total number of combinations. D. ​No, because the permutations rule would be used to determine the total number of combinations.

Answers

The correct answer is D. ​No because the permutations rule would be used to determine the total number of combinations.

Explanation:

The difference between a combination and a permutation is that in permutations the order is considered. This applies to the numbers in a lock because these need to be in order. Therefore, to analyze the permutations in a lock, the rule for permutations should be used. This includes the general formula P (n,r) =[tex]\frac{n!}{(n-r) !}[/tex]; in this, n is the number of objects and r refers to the objects used in a permutation. Thus, the term "combination" is inappropriate because this is a permutation, and the permutation rule should be used.

How can you change a rational number to a decimal? Can you give an exsample?

Answers

Answer:

1/2=0.5

Step-by-step explanation:

¼=0.25

¾=0.75

Please answer this question now in two minutes

Answers

Answer:

m∠C = 102°

Step-by-step explanation:

This diagram is a Quadrilateral inscribed in a circle

The first step is to determine what m∠B

is

The sum of opposite angles in an inscribed quadrilateral is equal to 180°

m∠D + m∠B = 180°

m∠B = 180° - m∠D

m∠B = 180° - 80°

m∠B = 100°

Second step is we proceed to determine the exterior angles of the circle

m∠ADC = 2 × m∠B

m∠ADC = 2 × 100°

m∠ADC = 200°

m∠ADC = m∠CD + m∠AD

m∠AD = m∠ADC - m∠CD

m∠AD = 200° - 116°

m∠AD = 84°

The third step is to determine m∠BAD

m∠BAD = m∠AD + m∠AB

m∠BAD = 84° + 120°

m∠BAD = 204°

The final step Is to determine what m∠C is

It is important to note that:

m∠BAD is Opposite m∠C

Hence

m∠C = 1/2 × m∠BAD

m∠C = 1/2 × 204

m∠C = 102°

Victor fue al mercado para comprar manzanas, naranjas y platanos; las naranjas costaron el doble de lo 1ue pago por las manzanas y los platanos costaron 8 pesos menos que pas manzanas, en total gasto 100 pesos. Determina el precio de las manzanas, naranjas y platanos

Answers

Answer:

El precio de las manzanas = 27 pesos

El precio de las naranjas = 54 pesos

El precio de las bananas = 19 pesos

Step-by-step explanation:

Los parámetros dados son;

El monto total gastado = 100 pesos

Sea el precio de las naranjas = x

Sea el precio de las manzanas = y

Sea el precio de los plátanos = z

La cantidad pagada por las naranjas = 2 · y = x

La cantidad pagada por los plátanos = y - 8 = z

Por lo tanto, tenemos;

La cantidad total gastada = La cantidad pagada por las naranjas + La cantidad pagada por las bananas + La cantidad pagada por las manzanas

∴ El monto total gastado = 100 pesos = 2 · y + y - 8 + y

100 = 4 · años - 8

4 · y = 100 + 8 = 108

y = 108/4 = 27

y = 27

De

z = y - 8 tenemos;

z = 27 - 8 = 19

De 2 · y = x, tenemos;

2 × 27 = x

x = 54

Por lo tanto;

El precio de las naranjas = 54 pesos

El precio de las manzanas = 27 pesos

El precio de los plátanos = 19 pesos.

A holiday company charters an aircraft to fly to Malta at
a cost of $22 000. It then sells 150 seats at $185 each and a
futher 35 seats at a 20% discount. Calculate the profit made
per seat if the plane has 200 seats.

Answers

Answer:

$54.65 profit per seat

Step-by-step explanation:

150(185) + 35(185)(.8) = 27,750 + 5,180 = 32,930 - 22,000 = 10,930

10,930/200 = $54.65 profit per seat

Answer:

$54.65

Step-by-step explanation:

First, we find the total amount made. This is easy:

(150 x 185) + (35 x .8(185)) =

27750 + 5180 =

32930

We then subtract the $22000, so the company makes a profit of 10930. There are 200 seats, so the profit made per seat is $54.65

Can someone help me with this please it’s algebra 2

Answers

Answer:

7 8 9

Step-by-step explanation:

what is the answer to 1/8=s-3/4

Answers

Answer:

7/8 =s

Step-by-step explanation:

1/8=s-3/4

Add 3/4

1/8 + 3/4 = s -3/4 +3/4

1/8 + 3/4 = s

Get a common denominator

1/8 + 3/4 *2/2 = s

1/8 + 6/8 =s

7/8 =s

1/8 = s - 3/4

1/8 = s -6/8 ( * 2/2)

7/8 = s

s = 7/8

solve this equation -2x+9=-5x-15

Answers

Answer:

x = -8

I hope this helps!

The answer would be x=-8

Evaluate the following expression. −8 × (−10) −7× 1/−1

Answers

Answer:

87

Step-by-step explanation:

[tex]-8\left(-10\right)-7 \times \frac{1}{-1}=87\\\\\mathrm{Apply\:rule}\:-\left(-a\right)=a\\\\=8\times \:10-7\times \frac{1}{-1}\\\\8\times \:10=80\\\\7\times \frac{1}{-1}=-7\\\\=80-\left(-7\right)\\\\\mathrm{Apply\:rule}\:-\left(-a\right)=a\\\\=80+7\\\\=87[/tex]

4x=24 solve equation

Answers

Answer:

x=6

Step-by-step explanation:

Rearrange:

Rearrange the equation by subtracting what is to the right of the equal sign from both sides of the equation :

                    4*x-(24)=0

Step by step solution :

STEP

1

:

Pulling out like terms

1.1     Pull out like factors :

  4x - 24  =   4 • (x - 6)

Equation at the end of step

1

:

STEP

2

:

Equations which are never true

2.1      Solve :    4   =  0

This equation has no solution.

A a non-zero constant never equals zero.

Solving a Single Variable Equation:

2.2      Solve  :    x-6 = 0

Add  6  to both sides of the equation :

                     x = 6

One solution was found :

x = 6

Answer:

x= 24/ 4

Step-by-step explanation:

You can simplify it

x= 6/1 which is x= 6

If 4SINB=3SIN(2A+B) :
Prove that:7COT(A+B)=COTA

Answers

Answer:

Step-by-step explanation:

Given the expression 4sinB = 3sin(2A+B), we are to show that the expression 7cot(A+B) = cotA

Starting with the expression

4sinB= 3sin(2A+B)

Let us re write angle B = (A + B) - A

and 2A + B = (A + B) + A

Substituting the derived expression back into the original expression ww will have;

4Sin{(A + B) - A } = 3Sin{(A + B)+ A}

From trigonometry identity;

Sin(D+E) = SinDcosE + CosDSinE

Sin(D-E) = SinDcosE - CosDSinE

Applying this in the expression above;

4{Sin(A+B)CosA - Cos(A+B)SinA} = 3{Sin(A+B)CosA + Cos(A+B)sinA}

Open the bracket

4Sin(A+B)CosA - 4Cos(A+B)SinA = 3Sin(A+B)CosA + 3Cos(A+B)sinA

Collecting like terms

4Sin(A+B)CosA - 3Sin(A+B)cosA = 3Cos(A+B)sinA + 4Cos(A+B)sinA

Sin(A+B)CosA = 7Cos(A+B)sinA

Divide both sides by sinA

Sin(A+B)CosA/sinA= 7Cos(A+B)sinA/sinA

Since cosA/sinA = cotA, the expression becomes;

Sin(A+B)cotA = 7Cos(A+B)

Finally, divide both sides of the resulting equation by sin(A+B)

Sin(A+B)cotA/sin(A+B) = 7Cos(A+B)/sin(A+B)

CotA = 7cot(A+B) Proved!

Joey intends to roll a six-sided number cube 100 times. What probability model can he use to predict whether or not each roll will give a result that is divisible by 3?

Answers

Options :

A. Each roll has a 0.117 probability of being divisible by 3.

B. Each roll has a 0.333 probability of being divisible by 3.

C. Each roll has a 0.5 probability of being divisible by 3. D. Each roll has a 0.667 probability of being divisible by 3.

Answer: B. Each roll has a 0.333 probability of being divisible by 3.

Step-by-step explanation:

Sample space for a six-sided number cube :

1, 2, 3, 4, 5, 6

Number of outcomes divisible by 3:

(3, 6) = 2

Probability of an event = Number of required outcomes / total number of possible items

Probability (getting a number divisible by 3):

(Number of outcomes divisible by 3 / total outcomes in sample space)

Probability (getting a number divisible by 3):

2 / 6 = 1/3

= 0.333

simpily 2^3×3^2=6^5​

Answers

Answer:

2^3×3^2=6^5​  equation is wrong because

2×2×2×3×3=72

6^5=6×6×6×6×6=36×36×6=7776

the two numbers are not equal

Mate, I think your question is wrong ! ;(

[tex]Corrected \\ Question...\\[/tex] (2^3)^2*(3^2)^3=6^5

Solve for x in the equation x squared + 11 x + StartFraction 121 Over 4 EndFraction = StartFraction 125 Over 4 EndFraction.

Answers

Answer:

Below

Step-by-step explanation:

● x^2 + 11x + 121/4 = 125/4

Substract 125/4 from both sides:

● x^2 + 11x + 121/4-125/4= 125/4 -125/4

● x^2 + 11x - (-4/4) = 0

● x^2 +11x -(-1) = 0

● x^2 + 11 x + 1 = 0

This is a quadratic equation so we will use the determinanant (b^2-4ac)

● a = 1

● b = 11

● c = 1

● b^2-4ac = 11^2-4*1*1 = 117

So this equation has two solutions:

● x = (-b -/+ √(b^2-4ac) ) / 2a

● x = (-11 -/+ √(117) ) / 2

● x = (-11 -/+ 3√(13))/ 2

● x = -0.91 or x = -10.9

Round to the nearest unit

● x = -1 or x = -11

The solutions are { -1,-11}

The solution of the equation x² + 11x + (121/4) = 125/4 will be 0.09 and negative 11.09.

What is the solution to the equation?

The distribution of weights to the variables involved that establishes the equilibrium in the calculation is referred to as a result.

The equation is given below.

x² + 11x + (121/4) = 125/4

Simplify the equation, then the equation will be

4x² + 44x + 121 = 125

4x² + 44x + 121 - 125 = 0

4x² + 44x - 4 = 0

x² + 11x - 1 = 0

We know that the formula, then we have

[tex]\rm x = \dfrac{-b \pm \sqrt {b^2 - 4ac}}{2a}[/tex]

The value of a = 1, b = 11, and c = -1. Then we have

[tex]\rm x = \dfrac{-11 \pm \sqrt {11^2 - 4 \times 1 \times (-1)}}{2 \times 1}\\\rm x = \dfrac{-11 \pm \sqrt {121 +4}}{2 }\\x = \dfrac{-11 \pm \sqrt {125}}{2 }[/tex]

Simplify the equation, then we have

x = (- 11 ± 11.18) / 2

x = (-11 - 11.18) / 2, (-11 + 11.18) / 2

x = -11.09, 0.09

The solution of the equation will be 0.09 and negative 11.09.

More about the solution of the equation link is given below.

https://brainly.com/question/545403

#SPJ6

please can someone help me solve this.. please help!!​

Answers

Step-by-step explanation:

Hello,

Firstly just look to triangle BDE,

Here, you will find that,

140° = y+80° {the exterior and opposite interior angle of a triangle is equal}.

or, y= 140°-80° {shifting 80° to another side and subtracting it.}

Therefore, the value of y is 60°.

now, let's simply work with line EB or EG. we get;

angle GEF + y=180° { being a linear pair}.

or, angle GEF + 60°= 180°

or, angle GEF = 180°-60°

Therefore, the value of angle GEF = 120°.

now, looking in triangle EFG, we get;

angle GEF + 35°+x= 180° { the sum of interior angle of a triangle is 180°}.

or, 120°+35°+ x= 180°

or, x= 180°- 155°

Therefore, the value of x is 25°.

now, lastly finding the value of "z"

We find that x= z {being vertical opposite angle}

or, z =25°

Therefore, the value of z is 25°.

So, the values are,

x=25°

y=60°

and z= 25°

Hope it helps...

70000000000x50000000000000

Answers

Answer:

Step-by-step explanation: Multiply

70000000000*50000000000000=3.5e+24

Evaluate the function below at x=5. Then, enter your solution. f(x)=3(2)^x

Answers

Answer:

Solution: f(5) = 96

Step-by-step explanation:

f(5) = 3(2)^5

f(5) = 3 (2 × 2 × 2 × 2 × 2)

f(5) = 3 (32)

f(5) = 96

Reduce any fractions to lowest terms. Don't round your answer, and don't use mixed fractions. 4x+4\leq9x+84x+4≤9x+8

Answers

Answer:

  x ≥ -4/5

Step-by-step explanation:

Maybe you want to solve ...

  4x+4 ≤ 9x +8

  0 ≤ 5x +4 . . . . . subtract 4x+4

  0 ≤ x +4/5 . . . . . divide by 5

  -4/5 ≤ x . . . . . . . subtract 4/5

Answer:

x ≥−4/5

Step-by-step explanation:

a blue die and a green die are rolled. find the probability that the blue and green are both less than 6​

Answers

Answer

5/6 maybe

Step-by-step explanation:

Other Questions
PLZ HELP THANKS! Find the equation of the line passing through the pair points (-8,6) (-9,-9). The equation of the line in the form is Ax+By=C. A box contains 20 equal-sized balls, numbered 1 to 20. Two balls are drawn at random simultaneously. What is the probability that the numbers on the two balls will differ by more than 2 Find the mean, median, and mode An 85-year old risk averse investor is not happy about the minimal return she is earning on her current investments. She is stressed about having enough income because her cost of living has been increasing by more than 10% annually. Her current portfolio composition consists of: 40% Money Market Fund 50% Bonds 10% Equities What changes should you suggest to her portfolio? A. Reduce the Money Market Fund allocation by 10% (to 30%) and put the released funds in commodities such as gold B. Reduce the Money Market Fund allocation by 30% (to 10%) and put the released funds in AAA-rated corporate bonds C. Liquidate the entire Money Market Fund allocation and put the released funds in Equities, bringing that allocation up to 50% D. Liquidate the entire Money Market Fund allocation and put the released funds in U.S. Treasury securities Read the excerpt from The Code Book. A theoretical breakthrough would be a fundamentally new way of finding Alice's private key. Alice's private key consists of p and q, and these are found by factoring the public key, N. The standard approach is to check each prime number one at a time to see if it divides into N, but we know that this takes an unreasonable amount of time. Which statement is best supported by this excerpt Which words could correctly replace the underlined words in the sentence below?Today was an exhausting day: I can't wait to relax this weekend.a tranquila peacefula gruelinga serene In a reversible reaction, the endothermic reaction absorbs ____________ the exothermic reaction releases.A. None of these, endothermic reactions release energyB. more energy thanC. the same amount of energy asD. less energy than Who supported the Nationalist Party in the Chinese Civil War? Germany the USSR the United States the rest of Asia The content of total body water in in adult women is * Describe the tone Grandma uses when she speaks. What kind of words help to create this tone? In a monster calls Test the claim that the proportion of people who own cats is significantly different than 80% at the 0.2 significance level. The null and alternative hypothesis would be:______. A. H0 : = 0.8 H 1 : 0.8 B. H0 : p 0.8 H 1 : p > 0.8 C. H0 : p = 0.8 H 1 : p 0.8 D. H0 : 0.8 H 1 : > 0.8 E. H0 : p 0.8 H 1 : p < 0.8 F. H0 : 0.8 H 1 : < 0.8 The test is:_____. a. left-tailed b. right-tailed c. two-tailed Based on a sample of 200 people, 79% owned cats.The test statistic is:______.The p-value is:_____. Based on this we:_____.A. Fail to reject the null hypothesis.B. Reject the null hypothesis. greg suffers from depression. His friend matthew also suffers from depression as well as conduct disorder. According to the theory of problem behavior syndrome, this means Oligopoly firms will seldom change prices but if one firm increases their price, others may follow if costs have ____________ . Find the approximate volume of this prism (Image down below) If there is a 1-degree change, coral reefs will become______? Which city enjoyed a long reign as the steel capital of the country due to its location at the confluence of three rivers Can you help Jorge organize the results into a two-way frequency table? Please answer this ASAP Candy is on sale for $0.75 each. You have a coupon for $0.25 off your total purchase. Write a function rule for the cost of n pieces of candy How did the English colonists profit from the slave trade?A. They sold the slaves to the colonies of other countries.B. They used slave labor to produce cash crops.C. They put well-trained slaves in their armies.D. They traded slaves to the natives for supplies. The function f(x) = 50(0.952)x, where x is the time in years, models a declining feral cat population. How many feral cats will there be in 9 years?